Hi pablourioste!
It's not quite that simple, but you're on the right track! The reason why (C) is correct and answer choice (A) is not stems from the fact that (C) properly conveys the conditional logic while (A) contains a mistaken reversal.
From the information in the stimulus, we find that the conclusion is in the last sentence in which it states "for a meeting to achieve maximum productivity,
then, it needs to have a clear time frame and be no more than 30 minutes long." So, we can create a conditional relationship from these terms. The word 'then' is a necessary condition indicator, so the following phrase will be the necessary condition.
Thus, our diagram looks like this: Maximum Productivity
Clear Time Frame &
More than 30 Minutes
Comparing this diagram to answer choice (A) and (C), we can see where (A) goes wrong. When diagrammed, answer choice (A) looks like:
More than 30 Minutes & Clear Time Frame
Maximum Productivity
So, you can see that (A) is effectively a Mistaken Reversal! It flipped the conditions improperly.
Turning to answer choice (C), we can diagram this statement to look like:
Maximum Productivity
Clear Time Frame &
More Than 30 Minutes. This matches our original diagram perfectly! Thus, we can see that it's a slightly rephrased version of the exact conclusion contained in the stimulus and accurately characterizes the conclusion.
I hope this helps
Kate